CBSE Questions for Class 12 Medical Physics Moving Charges And Magnetism Quiz 1 - MCQExams.com

A particle with a specific charge s is fired with a speed v towards a wall at a distance d, perpendicular to the wall. What minimum magnetic field must exist in this region for the particle not to hit the wall?
  • $$\frac{v}{sd}$$
  • $$\frac{2v}{sd}$$
  • $$\frac{v}{2sd}$$
  • $$\frac{v}{4sd}$$
The distance between the supply wires of an electric-mains is $$12cm$$. These wires experience $$4mg$$ weight per unit length. Current flowing in both the wire will be if they carry current in same direction.
  • zero
  • $$4.85A$$
  • $$4.85A$$
  • $$4.85\times {10}^{-4}A$$
An electron, a proton and an alpha particle having the same kinetic energy are moving in circular orbits of radii $$r_e, r_p, r_{\alpha}$$ respectively in a uniform magnetic field B. The relation between $$r_e, r_p, r_{\alpha}$$ is?
  • $$r_e < r_p < r_{\alpha}$$
  • $$r_e < r_{\alpha} < r_p$$
  • $$r_e > r_p = r_{\alpha}$$
  • $$r_e < r_p = r_{\alpha}$$

A particle of charge $$-16\times 10^{-18}$$ coulomb moving with velocity 10 $$\mathrm{m}\mathrm{s}^{-1}$$ along the x-axis enters a region where a magnetic field of induction $$\mathrm{B}$$ is along the $$\mathrm{y}$$ -axis, and an electric field of induction $$\mathrm{B}$$ is along the $$\mathrm{y}$$-axis, and an electric field of magnitude $$10^{4}\mathrm{V}/\mathrm{m}$$ is along the negative $$\mathrm{z}$$-axis. lf the charged particle continues moving along the $$\mathrm{x}$$-axis, the magnitude of $$\mathrm{B}$$ is
  • $$10^{3}\mathrm{W}\mathrm{b}/\mathrm{m}^{2}$$
  • $$10^{5}\mathrm{W}\mathrm{b}/\mathrm{m}^{2}$$
  • $$10^{16}\mathrm{W}\mathrm{b}/\mathrm{m}^{2}$$
  • $$10^{-3}\mathrm{W}\mathrm{b}/\mathrm{m}^{2}$$
Which of the following expressions are applicable to the moving coil galvanometer?
  • $$\displaystyle \overrightarrow{F_m}=q(\overrightarrow {V}\times\overrightarrow{B})$$
  • $$\displaystyle B=B_0\tan\theta$$
  • $$\displaystyle \overrightarrow {\tau}=\overrightarrow{M}\times\overrightarrow{B}$$
  • none of these
A charged particle moves through a magnetic field perpendicular to its direction. Then
  • the momentum changes but the kinetic energy is constant
  • both momentum and kinetic energy of the particle are not constant
  • both, momentum and kinetic energy of the particle are constant
  • kinetic energy changes but the momentum is constant
Two long conductors, separated by a distance $$d$$ carry currents $$I_1$$ and $$I_2$$ in the same direction. They exert a force $$F$$ on each other. Now the current in one of them is increased to two times and its direction is reversed. The distance is also increased to $$3d$$. The new value of the force between them is: 
  • $$-2F$$
  • $$\dfrac{F}{3}$$
  • $$\dfrac{-2F}{3}$$
  • $$\dfrac{-F}{3}$$
Two long straight parallel wires, carrying (adjustable) current $$I_1$$ and $$I_2$$, are kept at a distance d apart. If the force 'F' between the two wires is taken as 'positive' when the wires repel each other and 'negative' when the wires attract each other, the graph showing the dependence of 'F', on the product $$I_1I_2$$, would be
A particle of mass M and charge Q moving with velocity $$\vec v$$ describe a circular path of radius R when subjected to a uniform transverse magnetic field of induction B. The work done by the field when the particle completes one full circle is
  • $$\displaystyle \left ( \frac{Mv^2}{R} \right ) 2 \pi R$$
  • $$zero$$
  • $$BQ2 \pi R$$
  • $$BQv2 \pi R$$
A particle of mass $$\mathrm{M}$$ and charge $$\mathrm{Q}$$ moving with velocity $$\vec{V}$$ describes a circular path of radius $$\mathrm{R}$$ when subjected to a uniform transverse magnetic field of induction B. The work done by the field when the particle completes one full circle is
  • $$(\displaystyle \frac{\mathrm{M}\mathrm{v}^{2}}{\mathrm{R}})2\pi \mathrm{R}$$
  • zero
  • $$\mathrm{B}2\pi \mathrm{R}$$
  • $$\mathrm{B}\mathrm{v}2\pi \mathrm{R}$$
A rectangular loop of sides 10 cm and 5 cm carrying a current I and 12 A is placed in different orientations as shown in the figures above.
If there is a uniform magnetic field of 0.3 T in the positive z direction, in which orientations the loop would be in (i) stable equilibrium and (ii) unstable equilibrium?
299446_6326fabaeaa749398a9463bdb909e6c1.png
  • (A) and (B), respectively
  • (A) and (C), respectively
  • (B) and (D), respectively
  • (B) and (C), respectively
A loop carrying current $$I$$ lies in the $$ x$$-$$y$$ plane as shown in the figure. The unit vector $$\hat{k}$$ is coming out of the plane of the paper. The magnetic moment of the current loop is

28908.png
  • $$a^{2}I\hat{k}$$
  • $$(\displaystyle \frac{\pi}{2}+1)a^{2}I\hat{k}$$
  • $$-(\displaystyle \frac{\pi}{2}+1 )a^{2}I\hat{k}$$
  • $$(2\pi+1)a^{2}I\hat{k}$$

An electron traveling with a speed $$\mathrm{u}$$ along the positive $$\mathrm{x}$$-axis enters into a region of magnetic field where $$\mathrm{B}=-\mathrm{B}_{0}\hat{\mathrm{k}}(\mathrm{x}>0)$$. It comes out of the region with speed $$\mathrm{v}$$ then :

43675_ad7e1646df9044049eb627c365527c58.png
  • v = u at y > 0
  • v = u at y < 0
  • v > u at y > 0
  • v > u at y < 0
Two identical bar magnets are fixed with their centres at a distance d apart. A stationary charge Q is placed at P in between the gap of the two magnets at a distance D from the centre O as shown in the figure.
The force on the charge Q is

306659.png
  • $$zero$$
  • directed along OP
  • directed along PO
  • directed perpendicular to the plane of paper

A proton and alpha particle both enter a region of uniform magnetic field B, moving at right angles to the field B. If the radius of circular orbits for both the particles is equal and the kinetic energy acquired by proton is 1 MeV, the energy acquired by the alpha particle will be:

  • 1 Me V
  • 4 Me V
  • 0.5 MeV
  • 1.5 MeV
A bar magnet is hung by a thin cotton thread in a uniform horizontal magnetic field and is in equilibrium state. the energy required to rotate it by $$60^o$$ is $$W$$, Now the torque required to keep the magnet in this new position is.
  • $$\dfrac{2W}{\sqrt{3}}$$
  • $$\dfrac{W}{\sqrt{3}}$$
  • $$\sqrt{3} W$$
  • $$\dfrac{\sqrt{3}}{2} W$$
A short bar magnet of magnetic moment $$0.4  J  {T}^{-1}$$ is place in a uniform magnetic field of $$0.16  T$$. The magnet is stable equilibrium when the potential energy is
  • $$-0.064 J$$
  • Zero
  • $$-0.082 J$$
  • $$0.064$$
Current sensitivity of a moving coil galvanometer is $$5$$ div/mA and its voltage sensitivity (angular deflection per unit voltage applied) is $$20$$ div/V. The resistance of the galvanometer is?
  • $$250$$ $$\Omega$$
  • $$40$$ $$\Omega$$
  • $$500$$ $$\Omega$$
  • $$25$$ $$\Omega$$
A proton carrying $$1MeV$$ kinetic energy is moving in a circular path of radius $$R$$ in uniform magnetic field. What should be the energy of an $$\alpha$$-particle to describe a circle of same radius in the same field?
  • $$1MeV$$
  • $$0.5MeV$$
  • $$4MeV$$
  • $$2MeV$$
  • Both Assertion and Reason are correct and Reason is the correct explanation for Assertion
  • Both Assertion and Reason are correct but Reason is not the correct explanation for Assertion
  • Assertion is correct but Reason is incorrect
  • Both Assertion and Reason are incorrect

1633711_2c323659a9d7476c86eae0ab897a352c.PNG
  • Both Assertion and Reason are correct and Reason is the correct explanation for Assertion
  • Both Assertion and Reason are correct but Reason is not the correct explanation for Assertion
  • Assertion is correct but Reason is incorrect
  • Both Assertion and Reason are incorrect
An uncharged particle is moving with a velocity of $$\vec { v } $$ in non-uniform magnetic field as shown. Velocity $$\vec { v } $$ would be:
468108.PNG
  • maximum at $$A$$ and $$B$$
  • minimum at $$A$$ and $$B$$
  • maximum at $$M$$
  • same at all points
Two wires carrying
  • parallel current repel each other.
  • antiparallel current attract each other.
  • antiparallel current repel each other.
  • equal magnitudes of antiparallel current attract each other.
A current carrying small loop behaves like a small magnet. If $$A$$ be its area and $$M$$ its magnetic moment, the current in the loop will be
  • $$M/A$$
  • $$A/M$$
  • $$MA$$
  • $$Am^{2}$$
When a charged particle is acted on only by a magnetic force, its:
  • potential energy changes
  • its kinetic energy changes
  • total energy changes
  • energy does not change
When $$0.50\overset {\circ}{A}$$ X-rays strike a material, the photoelectrons from the $$k$$ shell are observed to move in a circle of radius $$23\ mm$$ in a magnetic field of $$2\times 10^{-2}$$ tesla acting perpendicularly to the direction of emission of photoelectrons. What is the binding energy of $$k-shell$$ electrons?
  • $$3.5\ keV$$
  • $$6.2\ keV$$
  • $$2.9\ keV$$
  • $$5.5\ keV$$
Magnetic moment of bar magnet is $$M$$. The work done in turning the magnet by $$90^o$$ in direction of magnetic field $$B$$ will be
  • Zero
  • $$\displaystyle\frac{1}{2}$$$$MB$$
  • $$3$$$$MB$$
  • $$MB$$
The correct figure showing Fleming's Left Hand Rule is :
Two like poles of strength $$m_{1}$$ and $$m_{2}$$ are far distance apart. The energy required to bring them $$r_{0}$$ distance apart is
  • $$\displaystyle \dfrac{\mu _{0}}{4\pi }\frac{m_{1}m_{2}}{r_{0}}$$
  • $$\displaystyle \dfrac{\mu _{0}}{8\pi }\frac{m_{1}m_{2}}{r_{0}}$$
  • $$0$$
  • $$\displaystyle \dfrac{\mu _{0}}{16\pi }\frac{m_{1}m_{2}}{r_{0}^2}$$

The work required to rotate a magnetic needle by 60$$^{0}$$ from equilibrium position in a uniform magnetic field is W. The torque required to hold it in that position is

  • $$\dfrac{\sqrt{3}}{2}$$W
  • W
  • $$\dfrac{W}{2}$$
  • $$\sqrt{3}W$$

A magnet of moment 4Am$$^{2}$$ is kept suspended in a magnetic field of induction $$5\times 10^{-5}T$$. The workdone in rotating it through 180$$^{0}$$ is

  • $$4\times 10^{-4}J$$
  • $$5\times 10^{-4}J$$
  • $$2\times 10^{-4}J$$
  • $$10^{-4}J$$

Assertion (A): A magnet remains stable, If it aligns itself with the field

Reason (R): The P.E. of a bar magnet is minimum, if it is parallel to magnetic field.

  • Both A and R are true and R is the correct explanation of A.
  • Both A and R are true and R is not correct explanation of A.
  • A is true, But R is false
  • A is false, But R is true
A bar magnet of magnetic moment $$2Am^{2}$$ is free  to rotate about a vertical axis passing through its center. The magnet is released from rest from east - west position. Then the KE of the magnet as it takes N-S position is
$$(B_{H}=25\mu T)$$

  • $$25 \mu J $$
  • $$50\mu J$$
  • $$100\mu J$$
  • $$12.5\mu J$$
A vertical straight conductor carries a current vertically upwards. A point $$P$$ lies to the east of it at a small distance and another point $$Q$$ lies to the west at the same distance the magnetic field at $$P$$ is :
  • greater than at $$Q$$
  • same as at $$Q$$
  • less than at $$Q$$
  • greater or less than at $$Q$$ depending upon the strength of current

The ratio of magnetic potential due to magnetic dipole at the end to that at the broad side on a position for the same distance from it is :

  • zero
  • $$\infty $$
  • 1
  • 2

Maximum P.E. of magnet of moment M situated in a magnetic field of induction B, is

  • $$\dfrac {1}{2}MB$$
  • $$\dfrac {M}{B}$$
  • $$2MB$$
  • $$MB$$
A short bar magnet placed with its axis at $$30^{o}$$ with a uniform external magnetic field of $$0.16\ T$$ experiences a torque of magnitude $$0.032 Nm$$  If the bar magnet is free to rotate, its potential energies when it is in stable and unstable equilibrium are respectively
  • $$-0.064\ J, +0.064\ J$$
  • $$-0.032\ J, +0.032\ J$$
  • $$+0.064\ J, -0.128\ J$$
  • $$0.032\ J, -0.032\ J$$

The dimensional formula for magnetic moment is :

  • $$[M^{0}L^{1}T^{0}A^{2}]$$
  • $$[M^{0}L^{2}T^{0}A^{1}]$$
  • $$[M^{0}L^{2}T^{0}A^{2}]$$
  • $$[M^{0}L^{0}T^{1}A^{1}]$$
A charge q is moving with a velocity v parallel to a magnetic field B. Force on the charge due to magnetic field is
  • qvB
  • qB/v
  • Zero
  • Bv/q
The following cases in which no force is exerted by a magnetic field on a charge is

  • moving with constant velocity
  • moving in a circle
  • at rest
  • moving along a curved path
The force acting on a charge q moving with a velocity V in a magnetic field of induction B is given by :
  • $$\dfrac{q}{\vec{V}\times \vec{B}}$$
  • $$\dfrac{\vec{V}\times \vec{B}}{q}$$
  • $$q(\vec{V}\times \vec{B})$$
  • $$(\vec{V}.\vec{B})q$$
A proton moving in a straight line enters a strong magnetic field along the field direction then its path and velocity will be

  • path is circular but speed constant
  • path is same but velocity increases
  • path is same and velocity remains constant
  • path is same and motion is retarded
A charge moving with velocity v in X - direction is subjected to a field of magnetic induction in the negative X direction. As a result the charge will

  • retard along X-axis
  • move along a helical path around X axis
  • remain unaffected
  • starts moving in a circular path
A current loop placed in a magnetic field behaves like a
  • magnetic dipole
  • magnetic substance
  • magnetic pole
  • non magnetic substance
The dipole moment of a current loop is independent of
  • current in the loop
  • number of turns
  • area of the loop
  • magnetic field in which it is situated
If an electron of velocity is $$2i+4j$$ is subjected to magnetic field of $$4k$$, then, for the electron ,
  • speed will change
  • path will change
  • velocity is Constant
  • momentum is Constant
If a charged particle is projected perpendicular to a uniform magnetic field, then
a) it revolves in circular path
b) its K.E. remains constant
c) its momentum remains constant
d) its path is spiral

  • only a, b are correct
  • only a, c are correct
  • only b, d are correct
  • only a, d are correct
There will be no force experienced if
  • two parallel wires carry currents in the same direction
  • two parallel wires carry currents in the opposite direction
  • a positive charge is projected between the pole pieces of a bar magnet
  • a positive charge is projected along the axis of a solenoid carrying current
The coil of the moving coil galvanometer is wound over an aluminium frame
  • because aluminium is a good conductor
  • because aluminium is very light.
  • because aluminium is comparatively cheaper
  • to provide electro-magnetic damping.
A circular current carrying coil has a radius r. Its magnetic dipole moment is proportional to

  • r
  • $$r^{2}$$
  • 1/r
  • 1/$$r^{2}$$
0:0:1


Answered Not Answered Not Visited Correct : 0 Incorrect : 0

Practice Class 12 Medical Physics Quiz Questions and Answers